1. Trang chủ
  2. » Địa lý

Định lý Wolstenholme và ứng dụng - Lê Phúc Lữ

8 162 0

Đang tải... (xem toàn văn)

THÔNG TIN TÀI LIỆU

Thông tin cơ bản

Định dạng
Số trang 8
Dung lượng 156,94 KB

Nội dung

ĐỊNH LÝ WOLSTENHOLME I) Một số kiến thức cơ bản.. Đa thức Lagrange. Định lý được chứng minh. Từ đây ta cũng chứng minh được định lý Wilson. II) Định lý Wolstenholme. Dễ thấy rằng tổng t[r]

(1)

Chủ đề

ĐỊNH LÝ WOLSTENHOLME I) Một số kiến thức

1 Cho p số nguyên tố, đó, ta có định lý quan trọng: (1)Wilson: (p1)! 1 0(mod ).p

(2)Fermat nhỏ: ap1 1(mod )p

với ( , )a p 1

Các định lý chứng minh dựa vào tính chất hệ thặng dư đầy đủ (TDĐĐ), thu gọn nghịch đảo số nguyên theo modulo p, cụ thể là:

2 Với số nguyên i 1,2, 3, ,p1 tồn số j 1,2, 3, ,p1 cho

1(mod )

ijp

(( , )a m 1 ax y chạy qua hệ TDĐĐ m x chạy qua hệ TDĐĐ m) Đặc biệt: i  1 j i     p j p

3 Định lý Viète:

Cho đa thức ( ) n 1 n 2 1 0

n n

P x a x a xa x a x a

      giả sử đa thức có n nghiệm r r1, , ,2 rn khác Khi đó, ta có:

1

1 2

1

1

1 1 2

( 1)

, ,

( 1) 1

n n

n n

n n

n n

j n

j i i

n n

a a

r r r r r r

a a

a

r r r r

a r r r

 

 

     

 

  

      

 



Ta biết đa thức P x( ) bậc n có tối đa n nghiệm

Điều phương trình đồng dư, tức P x( )0(mod )m với P x( ) có bậc m có tối đa m nghiệm (khơng có cặp nghiệm đồng dư với theo mod m) VD P x( )x3   x (mod 3) có nghiệm x 1(mod 3)

11

( ) (mod11)

P xx  x có đủ 11 nghiệm theo định lý Fermat nhỏ

4 Đồng dư thức phân số: Ta quy ước ký hiệu a c(mod )m

bd , a b c d, , , 

, 0,( , ) ( , )

b db md m

0(mod )

(2)

Chẳng hạn, thay viết ij 1(mod )p trên, ta viết j(mod )p i  Đa thức Lagrange

Xét đa thức f x( )(x 1)(x 2) (x  p 1) với p số nguyên tố Khai triển ra, ta có:

1 2

2

( ) p p ( 1)!

p

f x xa xa x a x p

      

Định lý Lagrange đa thức nói hệ số ai với i 1,2, ,p2 chia hết cho

p Thật vậy,

Xét đa thức P x( ) f x( )xp11, có bậc p2

Hơn P(1)P(2) P p(  1) 0(mod )p nên đa thức có p1 nghiệm, tức tất hệ số đa thức phải chia hết cho p

Suy ap2,ap3, , , ,(a a p2 1 1)! 1 phải chia hết cho p Định lý chứng minh Từ ta chứng minh định lý Wilson

II) Định lý Wolstenholme

Dạng Cho p số nguyên tố lớn 3, ta có:

2 2

1 1

0(mod )

1 2   (p1)  p

Chứng minh Sử dụng nghịch đảo số theo modulo p tổng tổng hoán vị số 1 ,2 , ,(2 p1)2 ta có

2 2

2 2

1 1 ( 1)(2 1)

( 1) 0(mod )

6

1 ( 1)

p p p

p p

p

 

          

Dạng Cho p số nguyên tố lớn 3, ta có:

2

1 1

0(mod )

1 2  p1  p

(3)

Chú ý 1 1 ( 1) 1 (mod 2)

1 p p p p

 

 

           

    nên sử dụng

lại đa thức Lagrange định lý Viète, ta cần chứng minh 0(mod )

ap Thật vậy, từ đồng thức:

1 2

2

( 1)( 2) ( 1) ( ) p p ( 1)!

p

x x x p f x xa xa x a x p

            ,

ta cho x  p vế trái ( 1)( 2) (   p 1) ( 1) (p1 p1)!(p1)! Do đó: ( )p 2( )p 2( )2 1

p

pa pa p pa

      

Dễ thấy vế trái chia hết cho p3 nên vế phải phải chia hết cho p3 hay p a2 1 Dạng Cho p số nguyên tố lớn 3, chứng minh

3

2 2(mod )

p p

Cp hay 2p 11 1(mod 3)

p

Cp

 

Chứng minh

Hai kết tương đương 2 (2 )! (2 1)! 2 11

! ! !( 1)!

p p

p p

p p

C C

p p p p

 

  

Nếu áp dụng đồng thức Vandermonde, ta chứng minh

2 2(mod )

p p

Cp Thật vậy, ta biết rằng:

0 2

2 ( ) ( ) ( )

p p

p p p p

CCC   C

Hai số hạng đầu cuối 1, số hạng chia hết cho p2

!

!( )!

i p

p C

i p i

 chia hết cho p với i 1,2, 3, ,p1 Để đưa từ modulo p2 sang p3, ta phải có cách hiệu

2

(2 1)! ( 1)( 2) ( 1)

1 1

!( 1)! ( 1)!

( 1)( 2) ( 1) ( 1)!

( 1)!

p p

p p p p p

C

p p p

p p p p p

p

    

    

 

     

Do p p3,( 1)!1 nên ta cần chứng minh

3

(4)

Tuy nhiên,

2

( ) ( 1)! p p

p p

f p p a pa pa p a p

 

       chia hết cho p3 nên định lý chứng minh

Định lý dạng khó bp b(mod 4) ap a

CC p có lời giải tổ hợp nên khơng trình bày

III) Các ví dụ áp dụng Bài

1) Cho p số nguyên tố lẻ, chứng minh rằng: 12 12 2 (mod )

2 (p 1) p

    

2) Cho p 3 số nguyên tố Đặt 1 ; ,

1

m m n

p n

     ( , )m n 1 Chứng minh p4 |mpn

Lời giải

1) Ta cần ghép số hạng thích hợp:

2

2 2

1

( ) ( )

p pi i p i i p i

 

  chia hết cho p với

1  i p

2) Ta có 1 1

1

m mp n

p n p np

     

Áp dụng định lý Wolstenholme cho vế trái mp n 0(mod )p2 np

 

n chia hết cho p nên np chia hết cho p2, suy mpn chia hết cho p4

Bài

1) Gọi f x( ) đa thức Lagrange Chứng minh f( x) f x( p)

2) (Mở rộng định lý Wolstenholme 3): Cho p 3 số nguyên tố m k, 

Chứng minh

1

1 (mod )

k

p k

mp

Cp

 

Lời giải

(5)

1

( 1)! ( 1)( 2) ( 1)

1 1

( 1)!

( 1)!( )!

( ) ( ) ( ) ( 1)!

1

( 1)! ( 1)! ( 1)!

k

k k k k

p

k mp

k k k

mp mp p mp p mp p p

C

p

p mp p

f mp p f mp f mp p

p p p

 

       

    

 

    

    

  

Biểu thức tử khai triển có dạng

1 2

2

( k p) ( k p) ( k) ( k)

p

mpa mpa mp a mp

       

p a2 2 nên dễ thấy biểu thức chia hết cho pk2

Ta thấy định lý nêu mạnh cách chứng minh, biến đổi tương tự định lý ban đầu

Bài (APMO 2006) Cho p3 số nguyên tố r số cách đặt p quân cờ lên bàn cờ hình vng có kích thước pp cho khơng có qn cờ hàng (nhưng cột) Chứng minh p5 | r

Lời giải

Số cách chọn p quân cờ tùy ý

p p

C

Số cách chọn thỏa mãn đề (trừ p cách không thỏa mãn) là:

p p

Cp

Ta cần chứng minh

5 0(mod ) p

p

C  p p , mà

2

2 2

1

2

( )! ( 1)!

!( )! ( 1)!( )!

p p

p p

p p p

C pC

p p p p p p p

 

 

  

    nên ta đưa

2

1

1 1(mod )

p p

Cp

 

Áp dụng định lý Wolstenholme mở rộng, ta có đpcm Bài 4.

1) Cho số nguyên x y, thỏa mãn x 0(mod ),p y 1(mod )p , hỏi xy ?(mod )p2 , p số nguyên tố

2) (Putnam 1980) Cho p số nguyên tố, chứng minh

0

2 (mod )

p

i i p

p p i i

C Cp

 

Lời giải

(6)

2 (mod )

xyx p

2) Ta có: với i1,2, 3, ,p1 Cpi chia hết cho p

( )! ( 1)( 2) ( ) ( 1)( 2) ( ) !

1 1

! ! ! !

i p i

p i p p p i p p p i i

C

i p i i

       

     

Dễ thấy tử số biểu thức cuối chia hết cho p nên i 1(mod 2) p i

C   p Với i 0 C Cp0 p0 1 với i p p 2p 2(mod 2)

p p

C Cp theo định lý Wolstenholme Từ suy

1

2

0 1

1 2 1(mod )

p p p p

i i i i i i p

p p i p p i p p

i i i i

C C C C C C p

 

 

   

         

   

Bài 5. (KHTN 2010) Cho dãy số ( )un xác định công thức:

1

3

1 4

0,

(4 2)

,

( 1) ( 1)

n n

u

n n n

u u n

n n

  

  

   

  



Chứng minh dãy số chứa vô số số hạng nguyên dương

Lời giải Ta có: (n 1)4un1 (4n 2)n u3 n 3n 1, đặt vnn u3 n

1

(n 1)vn (4n 2)vn 3n 1 (n 1)(vn 1)(4n2)(vn 1)

Lại đặt wnvn 1 1

1

n n

n

w v

n

 

 Ta tính v1 0,w1 1 nên

1

1

4 4 (2 1)!! (2 1)!!

1 ! !

n n

n n n

n n n n n

w w w w

n n n n n

 

 

    

     

 ,

trong (2n 1)!! tích số lẻ từ đến 2n 1, (2 )!n tích số chẵn từ đến n Ta biến đổi tiếp:

1

1

2 (2 1)!! ( 1)!(2 1)!! (2 2)!!(2 1)!! (2 1)!

! !( 1)! !( 1)! !( 1)!

n n

n n

n n n n n n

C

n n n n n n n

 

 

     

   

  

Suy

1

1

2

1

n

n n n

n n n n n

C

w C v C u

n

  

 

(7)

Theo định lý Wolstenholme up , mà có vơ số số ngun tố nên có vơ số số hạng dãy số nguyên Ta có đpcm

Bài 1) (IMO 1979) Chứng minh 1 (mod1979)

1  2 1319 

2) Tổng qt hóa tốn Lời giải

1) Chú ý 1979 nguyên tố, ta cần tìm cách tách ghép thích hợp

989

1

1 1 1 1 1 1

1 1319 1319 1318

1 1 1 1

1 1319 659

1 1 1 1

(mod1979)

1 1319 1320 1978 ii 1979 i

 

 

              

 

 

 

          

 

 

 

           

 

2) Với ý tưởng tách ghép trên, ta thấy rằng:

- Nếu p 3k 2 số nguyên tố 1 (mod ) 1   2 2k 1 p

- Nếu p 3k 1 số nguyên tố 1 (mod ) 1   2 2kp Bài 7. Cho p số nguyên tố

1) Chứng minh

1

1 ( 1)

(mod ) k

k p

C p

p k

 với 1  k p

2) (Putnam 1996): Cho p 5 số nguyên tố đặt

p k    

 

2

1 (mod ) k

i p i

C p

Lời giải

1) Chỉ cần biến đổi trực tiếp có đpcm 2) Từ 1, ta thấy k ( 1)k (mod 2)

p

p

C p

k

 

(8)

Bài 8. (IMO Shortlist 2013) Cho p số nguyên tố lẻ Đặt

2

,

1

p a

a a a

S a p         

Đặt S3 S4 3S2 m n

   với m n, 

Chứng minh p m

Lời giải Áp dụng bổ đề trên, ta có ( 1) (mod )

k k k k p a a C p k p

  Suy

1

1

1

1 ( 1)

( 1) ( 1) (mod )

k k p p

p p p

k k p k k k

a p p

k k k

a a a a

S C a C a p

k p p p

                           Do

3

2 3 3(2 ) (2 2)

3 (mod )

p p p p p p

S S S p

p p

       

    

Theo định lý Fermat nhỏ 2p 2(mod )p

nên (2p 2)2

p  hay

2 (2 2) 0(mod ) p p p  

Từ suy đpcm Đây tốn N7 đề IMO Shortlist 2011 Ý tưởng khó phát sử dụng bổ đề

Bài 9. (Chọn đội tuyển Nghệ An 2012) Cho p3 số nguyên tố đặt M 1,2, 3, ,p Với số nguyên dương k thỏa 1 k p, xét tập hợp Ek AM A: k đại

lượng (min max )

k

k A E

x A A

   Chứng minh

1

3

0 (mod )

p

k k p k

x C p

 

Lời giải Trước hết, với i, ta đếm có tập hợp có k phần tử nhận i - Nếu i 1 có

1 k p

C

 tập hợp - Nếu i 2 có

1 k p

C

 tập hợp

Từ suy ra, có tất k p i

C

 tập hợp nhận i phần tử nhỏ nhất, p  i k

Tương tự, có tất 1 k i

C

 tập hợp nhận i phần tử lớn nhất, ik Do đó:

1

1

1 1

1 1

(min max )

( ) ( 1) ( 1)

k

k k

k p i i

A E p i k i k

k k k k

i i i p

i k i k i k

x A A iC iC

p i C iC p C p C

                                   

Từ suy ra:  

1

3

1

( 1) ( 1) 0(mod )

p p

k k k p

k p p p p

k k

x C p C C p C p

 

 

     

  theo định lý

Ngày đăng: 08/02/2021, 07:54

TỪ KHÓA LIÊN QUAN

TÀI LIỆU CÙNG NGƯỜI DÙNG

TÀI LIỆU LIÊN QUAN

w